Can we generate random numbers using irrational numbers like π and e? Planned maintenance scheduled April 17/18, 2019 at 00:00UTC (8:00pm US/Eastern) Announcing the arrival of Valued Associate #679: Cesar Manara Unicorn Meta Zoo #1: Why another podcast?Perfect random number generation using normal numbersIs rejection sampling the only way to get a truly uniform distribution of random numbers?Is there a software algorithm that can generate a non-deterministic chaos pattern?Can a transcendental number like $e$ or $pi$ be compressed as not algorithmically random?Recurrence relations that do not have a closed form solutionHow to detect repeating random numbers?Generate a Random Diagonally Dominant MatrixCan a relatively small subset of random numbers be permuted and reused and still guarantee good expected running time for an algorithm like quicksort?Perfect random number generation using normal numbersCreating Random Number Generator with rangeHow to generate evenly distributed random numbers from the tics of a Geiger Counter?

Check which numbers satisfy the condition [A*B*C = A! + B! + C!]

Doubts about chords

Gastric acid as a weapon

Why did the IBM 650 use bi-quinary?

Can inflation occur in a positive-sum game currency system such as the Stack Exchange reputation system?

Did Kevin spill real chili?

Can a non-EU citizen traveling with me come with me through the EU passport line?

If 'B is more likely given A', then 'A is more likely given B'

Why don't the Weasley twins use magic outside of school if the Trace can only find the location of spells cast?

Should gear shift center itself while in neutral?

What is the correct way to use the pinch test for dehydration?

List *all* the tuples!

Using et al. for a last / senior author rather than for a first author

"Seemed to had" is it correct?

Can Pao de Queijo, and similar foods, be kosher for Passover?

How can I make names more distinctive without making them longer?

Why is black pepper both grey and black?

Is above average number of years spent on PhD considered a red flag in future academia or industry positions?

I am not a queen, who am I?

What's the purpose of writing one's academic bio in 3rd person?

G-Code for resetting to 100% speed

What are the motives behind Cersei's orders given to Bronn?

Is there a concise way to say "all of the X, one of each"?

Is it true that "carbohydrates are of no use for the basal metabolic need"?



Can we generate random numbers using irrational numbers like π and e?



Planned maintenance scheduled April 17/18, 2019 at 00:00UTC (8:00pm US/Eastern)
Announcing the arrival of Valued Associate #679: Cesar Manara
Unicorn Meta Zoo #1: Why another podcast?Perfect random number generation using normal numbersIs rejection sampling the only way to get a truly uniform distribution of random numbers?Is there a software algorithm that can generate a non-deterministic chaos pattern?Can a transcendental number like $e$ or $pi$ be compressed as not algorithmically random?Recurrence relations that do not have a closed form solutionHow to detect repeating random numbers?Generate a Random Diagonally Dominant MatrixCan a relatively small subset of random numbers be permuted and reused and still guarantee good expected running time for an algorithm like quicksort?Perfect random number generation using normal numbersCreating Random Number Generator with rangeHow to generate evenly distributed random numbers from the tics of a Geiger Counter?










17












$begingroup$


Irrational numbers like $pi$, $e$ and $sqrt2$ have a unique and non-repeating sequence after the decimal point. If we extract the $n$-th digit from such numbers (where $n$ is the number of times the method is called) and make a number with the digits as it is, should we not get a perfect random number generator? For example, if we're using $sqrt2$, $e$ and $pi$, the first number is 123, second one is 471, the next one is 184 and so on.










share|cite|improve this question









New contributor




Abhradeep Sarkar is a new contributor to this site. Take care in asking for clarification, commenting, and answering.
Check out our Code of Conduct.







$endgroup$







  • 30




    $begingroup$
    You have a strange definition of "random" in your head. "Random" means "unpredictable". How is your sequence unpredictable? What definition of "random" do you have in mind? Perhaps what you are calling "random" has another name.
    $endgroup$
    – Eric Lippert
    Apr 10 at 17:55







  • 5




    $begingroup$
    Note the spigot algorithm can be used to generate any hex digit in pi, without having to generate prior digits.
    $endgroup$
    – rcgldr
    Apr 11 at 0:26






  • 8




    $begingroup$
    @EricLippert Aren't all pseudorandom number generators predictable?
    $endgroup$
    – Federico Poloni
    Apr 11 at 6:23






  • 5




    $begingroup$
    The term has come up a few times: this is a "psuedo random number" not a "random number." It's a number generated algorithmically (so not random), but which has many desirable properties that random numbers have. Another algorithm is the "NYC phonebook" algorithm, where you go down the list of phone numbers, alphabetically, and take the last digit from each of them. Not random, but pseudorandom with some rather nice statistical behaviors!
    $endgroup$
    – Cort Ammon
    Apr 11 at 7:10






  • 5




    $begingroup$
    "Pseudo" means "similar to but not". So pseudo random numbers are similar to, but not random numbers. So I'm not following your train of thought here. Now, crypto-strength PRNGs have the desirable property that if the internal state is unknown to the attacker, no statistical test we possess can distinguish a crypto PRNG from a true RNG, and that includes their lack of predictability. But the digits of pi do not have that property; they are highly predictable.
    $endgroup$
    – Eric Lippert
    Apr 11 at 12:56















17












$begingroup$


Irrational numbers like $pi$, $e$ and $sqrt2$ have a unique and non-repeating sequence after the decimal point. If we extract the $n$-th digit from such numbers (where $n$ is the number of times the method is called) and make a number with the digits as it is, should we not get a perfect random number generator? For example, if we're using $sqrt2$, $e$ and $pi$, the first number is 123, second one is 471, the next one is 184 and so on.










share|cite|improve this question









New contributor




Abhradeep Sarkar is a new contributor to this site. Take care in asking for clarification, commenting, and answering.
Check out our Code of Conduct.







$endgroup$







  • 30




    $begingroup$
    You have a strange definition of "random" in your head. "Random" means "unpredictable". How is your sequence unpredictable? What definition of "random" do you have in mind? Perhaps what you are calling "random" has another name.
    $endgroup$
    – Eric Lippert
    Apr 10 at 17:55







  • 5




    $begingroup$
    Note the spigot algorithm can be used to generate any hex digit in pi, without having to generate prior digits.
    $endgroup$
    – rcgldr
    Apr 11 at 0:26






  • 8




    $begingroup$
    @EricLippert Aren't all pseudorandom number generators predictable?
    $endgroup$
    – Federico Poloni
    Apr 11 at 6:23






  • 5




    $begingroup$
    The term has come up a few times: this is a "psuedo random number" not a "random number." It's a number generated algorithmically (so not random), but which has many desirable properties that random numbers have. Another algorithm is the "NYC phonebook" algorithm, where you go down the list of phone numbers, alphabetically, and take the last digit from each of them. Not random, but pseudorandom with some rather nice statistical behaviors!
    $endgroup$
    – Cort Ammon
    Apr 11 at 7:10






  • 5




    $begingroup$
    "Pseudo" means "similar to but not". So pseudo random numbers are similar to, but not random numbers. So I'm not following your train of thought here. Now, crypto-strength PRNGs have the desirable property that if the internal state is unknown to the attacker, no statistical test we possess can distinguish a crypto PRNG from a true RNG, and that includes their lack of predictability. But the digits of pi do not have that property; they are highly predictable.
    $endgroup$
    – Eric Lippert
    Apr 11 at 12:56













17












17








17


3



$begingroup$


Irrational numbers like $pi$, $e$ and $sqrt2$ have a unique and non-repeating sequence after the decimal point. If we extract the $n$-th digit from such numbers (where $n$ is the number of times the method is called) and make a number with the digits as it is, should we not get a perfect random number generator? For example, if we're using $sqrt2$, $e$ and $pi$, the first number is 123, second one is 471, the next one is 184 and so on.










share|cite|improve this question









New contributor




Abhradeep Sarkar is a new contributor to this site. Take care in asking for clarification, commenting, and answering.
Check out our Code of Conduct.







$endgroup$




Irrational numbers like $pi$, $e$ and $sqrt2$ have a unique and non-repeating sequence after the decimal point. If we extract the $n$-th digit from such numbers (where $n$ is the number of times the method is called) and make a number with the digits as it is, should we not get a perfect random number generator? For example, if we're using $sqrt2$, $e$ and $pi$, the first number is 123, second one is 471, the next one is 184 and so on.







randomized-algorithms randomness random-number-generator






share|cite|improve this question









New contributor




Abhradeep Sarkar is a new contributor to this site. Take care in asking for clarification, commenting, and answering.
Check out our Code of Conduct.











share|cite|improve this question









New contributor




Abhradeep Sarkar is a new contributor to this site. Take care in asking for clarification, commenting, and answering.
Check out our Code of Conduct.









share|cite|improve this question




share|cite|improve this question








edited Apr 10 at 13:17









dkaeae

2,36211022




2,36211022






New contributor




Abhradeep Sarkar is a new contributor to this site. Take care in asking for clarification, commenting, and answering.
Check out our Code of Conduct.









asked Apr 10 at 12:53









Abhradeep SarkarAbhradeep Sarkar

9716




9716




New contributor




Abhradeep Sarkar is a new contributor to this site. Take care in asking for clarification, commenting, and answering.
Check out our Code of Conduct.





New contributor





Abhradeep Sarkar is a new contributor to this site. Take care in asking for clarification, commenting, and answering.
Check out our Code of Conduct.






Abhradeep Sarkar is a new contributor to this site. Take care in asking for clarification, commenting, and answering.
Check out our Code of Conduct.







  • 30




    $begingroup$
    You have a strange definition of "random" in your head. "Random" means "unpredictable". How is your sequence unpredictable? What definition of "random" do you have in mind? Perhaps what you are calling "random" has another name.
    $endgroup$
    – Eric Lippert
    Apr 10 at 17:55







  • 5




    $begingroup$
    Note the spigot algorithm can be used to generate any hex digit in pi, without having to generate prior digits.
    $endgroup$
    – rcgldr
    Apr 11 at 0:26






  • 8




    $begingroup$
    @EricLippert Aren't all pseudorandom number generators predictable?
    $endgroup$
    – Federico Poloni
    Apr 11 at 6:23






  • 5




    $begingroup$
    The term has come up a few times: this is a "psuedo random number" not a "random number." It's a number generated algorithmically (so not random), but which has many desirable properties that random numbers have. Another algorithm is the "NYC phonebook" algorithm, where you go down the list of phone numbers, alphabetically, and take the last digit from each of them. Not random, but pseudorandom with some rather nice statistical behaviors!
    $endgroup$
    – Cort Ammon
    Apr 11 at 7:10






  • 5




    $begingroup$
    "Pseudo" means "similar to but not". So pseudo random numbers are similar to, but not random numbers. So I'm not following your train of thought here. Now, crypto-strength PRNGs have the desirable property that if the internal state is unknown to the attacker, no statistical test we possess can distinguish a crypto PRNG from a true RNG, and that includes their lack of predictability. But the digits of pi do not have that property; they are highly predictable.
    $endgroup$
    – Eric Lippert
    Apr 11 at 12:56












  • 30




    $begingroup$
    You have a strange definition of "random" in your head. "Random" means "unpredictable". How is your sequence unpredictable? What definition of "random" do you have in mind? Perhaps what you are calling "random" has another name.
    $endgroup$
    – Eric Lippert
    Apr 10 at 17:55







  • 5




    $begingroup$
    Note the spigot algorithm can be used to generate any hex digit in pi, without having to generate prior digits.
    $endgroup$
    – rcgldr
    Apr 11 at 0:26






  • 8




    $begingroup$
    @EricLippert Aren't all pseudorandom number generators predictable?
    $endgroup$
    – Federico Poloni
    Apr 11 at 6:23






  • 5




    $begingroup$
    The term has come up a few times: this is a "psuedo random number" not a "random number." It's a number generated algorithmically (so not random), but which has many desirable properties that random numbers have. Another algorithm is the "NYC phonebook" algorithm, where you go down the list of phone numbers, alphabetically, and take the last digit from each of them. Not random, but pseudorandom with some rather nice statistical behaviors!
    $endgroup$
    – Cort Ammon
    Apr 11 at 7:10






  • 5




    $begingroup$
    "Pseudo" means "similar to but not". So pseudo random numbers are similar to, but not random numbers. So I'm not following your train of thought here. Now, crypto-strength PRNGs have the desirable property that if the internal state is unknown to the attacker, no statistical test we possess can distinguish a crypto PRNG from a true RNG, and that includes their lack of predictability. But the digits of pi do not have that property; they are highly predictable.
    $endgroup$
    – Eric Lippert
    Apr 11 at 12:56







30




30




$begingroup$
You have a strange definition of "random" in your head. "Random" means "unpredictable". How is your sequence unpredictable? What definition of "random" do you have in mind? Perhaps what you are calling "random" has another name.
$endgroup$
– Eric Lippert
Apr 10 at 17:55





$begingroup$
You have a strange definition of "random" in your head. "Random" means "unpredictable". How is your sequence unpredictable? What definition of "random" do you have in mind? Perhaps what you are calling "random" has another name.
$endgroup$
– Eric Lippert
Apr 10 at 17:55





5




5




$begingroup$
Note the spigot algorithm can be used to generate any hex digit in pi, without having to generate prior digits.
$endgroup$
– rcgldr
Apr 11 at 0:26




$begingroup$
Note the spigot algorithm can be used to generate any hex digit in pi, without having to generate prior digits.
$endgroup$
– rcgldr
Apr 11 at 0:26




8




8




$begingroup$
@EricLippert Aren't all pseudorandom number generators predictable?
$endgroup$
– Federico Poloni
Apr 11 at 6:23




$begingroup$
@EricLippert Aren't all pseudorandom number generators predictable?
$endgroup$
– Federico Poloni
Apr 11 at 6:23




5




5




$begingroup$
The term has come up a few times: this is a "psuedo random number" not a "random number." It's a number generated algorithmically (so not random), but which has many desirable properties that random numbers have. Another algorithm is the "NYC phonebook" algorithm, where you go down the list of phone numbers, alphabetically, and take the last digit from each of them. Not random, but pseudorandom with some rather nice statistical behaviors!
$endgroup$
– Cort Ammon
Apr 11 at 7:10




$begingroup$
The term has come up a few times: this is a "psuedo random number" not a "random number." It's a number generated algorithmically (so not random), but which has many desirable properties that random numbers have. Another algorithm is the "NYC phonebook" algorithm, where you go down the list of phone numbers, alphabetically, and take the last digit from each of them. Not random, but pseudorandom with some rather nice statistical behaviors!
$endgroup$
– Cort Ammon
Apr 11 at 7:10




5




5




$begingroup$
"Pseudo" means "similar to but not". So pseudo random numbers are similar to, but not random numbers. So I'm not following your train of thought here. Now, crypto-strength PRNGs have the desirable property that if the internal state is unknown to the attacker, no statistical test we possess can distinguish a crypto PRNG from a true RNG, and that includes their lack of predictability. But the digits of pi do not have that property; they are highly predictable.
$endgroup$
– Eric Lippert
Apr 11 at 12:56




$begingroup$
"Pseudo" means "similar to but not". So pseudo random numbers are similar to, but not random numbers. So I'm not following your train of thought here. Now, crypto-strength PRNGs have the desirable property that if the internal state is unknown to the attacker, no statistical test we possess can distinguish a crypto PRNG from a true RNG, and that includes their lack of predictability. But the digits of pi do not have that property; they are highly predictable.
$endgroup$
– Eric Lippert
Apr 11 at 12:56










6 Answers
6






active

oldest

votes


















13












$begingroup$

The most obvious disadvantage is the unnecessary complexity of PRNG algorithms based on irrational numbers. They require much more computations per generated digit than, say, an LCG; and this complexity typically grows as you go further in the sequence. Calculating 256 bits of π at the two-quadrillionth bit took 23 days on 1000 computers (back in 2010) - a rather prohibitive complexity for an RNG.






share|cite|improve this answer









$endgroup$




















    44












    $begingroup$

    For any reasonable definition of perfect, the mechanism you describe is not a perfect random number generator.



    • Non-repeating isn't enough. The decimal number $0.101001000100001dots$ is non-repeating but it's a terrible generator of random digits, since the answer is "always" zero, occasionally one, and never anything else.


    • We don't actually know if every digit occurs equally often in the decimal expansion of $pi$ or $mathrme$ (though we suspect they do).


    • In many situations, we require random numbers to be unpredictable (indeed, if you asked a random person what "random" means, they'd probably say something about unpredictability). The digits of well-known constants are totally predictable.


    • We usually want to generate random numbers reasonably quickly, but generating successive digits of mathematical constants tends to be quite expensive.


    • It is, however, true that the digits of $pi$ and $mathrme$ look statistically random, in the sense that every possible sequence of digits seems to occur about as often as it should. So, for example, each digit does occur very close to one time in ten; each two-digit sequence very close to one in a hundred, and so on.






    share|cite|improve this answer











    $endgroup$








    • 11




      $begingroup$
      For the third point, there must be some sort of 'secret' input to your generation process for it to be unpredictable (the generation process itself should be deterministic if we don't want to rely on yet another random number generator.). This extra input is often called a seed.
      $endgroup$
      – Discrete lizard
      Apr 10 at 14:01






    • 6




      $begingroup$
      @Discretelizard This is true but there's not much scope for seeding beyond "return successive digits starting with position $s$." By the time you've seen $2log s$ digits, that sequence occurs only a few times within the first $s^2$ digits of $pi$, so it's unique within the first $s$ digits with high probability and you know the seed.
      $endgroup$
      – David Richerby
      Apr 10 at 16:15







    • 2




      $begingroup$
      @Barmar: At that point you have to ask whether this technique is really more performant (and more space-efficient) than a "standard" PRNG would be.
      $endgroup$
      – Kevin
      Apr 11 at 0:53







    • 2




      $begingroup$
      The digits of pi or e are completely unpredictable, especially since the viewer / recipient / code breaker etc has no idea how far along in the sequence you already are. If you start at digit number 237423 of the sequence, it will take so long to figure out, as to be random.
      $endgroup$
      – DaveBoltman
      Apr 11 at 6:12






    • 10




      $begingroup$
      @DaveBoltman If we're not doing something like cryptography, nobody's going to care enough to bother figuring it out. If we are doing cryptography, it's a standard assumption that your adversary knows what algorithm you're using which, in this case, includes what irrational number the sequence is coming from and how you're choosing the digits, except for any parameter such as "start at digit $s$". If the adversary doesn't know what number you're using then, sure, the next digit could be literally anything, but then they guess it's $sqrttextmy birthday$ and the game's up.
      $endgroup$
      – David Richerby
      Apr 11 at 8:52


















    28












    $begingroup$

    It is cryptographically useless because an adversary can predict every single digit. It is also very time consuming.






    share|cite|improve this answer









    $endgroup$








    • 9




      $begingroup$
      OP never mentions cryptography...
      $endgroup$
      – AnoE
      Apr 10 at 16:19






    • 13




      $begingroup$
      @AnoE So? That this process would be cryptographically useless is still relevant because crypto is an avid user of randomness. If you bring up the devices /dev/random and /dev/urandom someone will invariably bring up cryptography.
      $endgroup$
      – Greg Schmit
      Apr 10 at 16:31






    • 6




      $begingroup$
      You would be amazed at how useless cryptographic security is in real time PRNG generation. irrational numbers are often used in GPU PRNGs. There are a lot of applications where how "secure" your PRNG is simply irrelevant. What matters in something like coherent noise generation is the quality of distribution and how often your period repeats, and correlation effects due to adjacent seeds (which would require avalanche mixers to fix). Quite honestly your answer is wrong, doesn't belong here, and should probably be deleted.
      $endgroup$
      – opa
      Apr 10 at 17:44







    • 6




      $begingroup$
      This is anot an answer to the question. Note the OP of the linked question uses random numbers for seeding a monte carlo analysis. An update to address the question asked should be considered. mathoverflow.net/questions/26942/…
      $endgroup$
      – CramerTV
      Apr 10 at 17:47







    • 8




      $begingroup$
      Certainly there are many applications where PRNGs don't need to be cryptographically secure. But OP didn't ask if it was useful for some purposes, they asked if this method was a "perfect RNG". While they haven't clarified what they mean by "perfect", the fact that it's unsuitable for one of the major uses of RNGs seems very relevant to answering that question.
      $endgroup$
      – Geoffrey Brent
      Apr 11 at 2:08



















    6












    $begingroup$

    (updated after many people pointed out that random number generator is not the same thing as a single normal sequence)



    If you ask whether you can get a normal sequence out of $pi$ (i.e., all numbers appear uniformly), then there are several answers on mathoverflow. For example, the answer about Distribution of the digits of Pi says:




    ...it is believed that $pi$ is a normal number (~uniform distribution of every digits sequence).




    For digit distribution data, see e.g. http://www.eveandersson.com/pi/precalculated-frequencies or https://thestarman.pcministry.com/math/pi/RandPI.html (first 1000 digits):



    enter image description here



    At mathoverflow, there are also nice answers at:



    • What is the state of our ignorance about the normality of pi?

    • Does pi contain 1000 consecutive zeroes?





    share|cite|improve this answer











    $endgroup$








    • 3




      $begingroup$
      If you believe the question is a duplicate, then why are you answering it? You should simply flag it, not reinforce undesired posting behavior.
      $endgroup$
      – dkaeae
      Apr 10 at 16:11






    • 8




      $begingroup$
      @dkaeae There is no support for duplicates of questions on other sites. Furthermore, the same question on different sites can get different answers. In this case, a site such as Mathematics might not give much consideration to security concerns. See also this answer. Do note that we discourage asking the same question on multiple sites at the same time, since this tends to lead to wasted efforts. But the same question by different persons at different times on different sites is usually ok.
      $endgroup$
      – Discrete lizard
      Apr 10 at 17:01






    • 6




      $begingroup$
      Unfortunately, just because a number is normal doesn't mean that outputting its digits gives you a good RNG. The outputs of such a RNG are still entirely predictable. Whether that's acceptable might depend on the application. So, I don't think it's quite as simple as saying "pi is normal, case closed".
      $endgroup$
      – D.W.
      Apr 10 at 18:06







    • 2




      $begingroup$
      That is just emperical observation for first few digits? What is to be meant by that?
      $endgroup$
      – marshal craft
      Apr 11 at 6:23






    • 1




      $begingroup$
      @D.W. I mentioned that I intend to use a combination of numbers like π and e. And please say how the output will be predictable if we do not know how far down the sequence the generator has gone?
      $endgroup$
      – Abhradeep Sarkar
      Apr 11 at 8:19


















    1












    $begingroup$

    In general, this approach does not work: "randomness" does not mean that you get a lot of different digits, but there are other aspects as well. For example, a classic test is to see if all two-digit, or three-digit etc. combinations occur with the same frequency. This would be a very simple test, which can rule out obvious non-random results, but is still by far too simplistic to check for really random behaviour.



    See the Wikipedia page about Randomness Tests as a collection of links to primary sources regarding this. They do mention a good amount of quite complicated-sounding concepts; it is it not so important to go into deep detail about this - but it is clear that it is not intuitively possible to declare a specific number to be a good source for such digits.



    On a positive note: For a specific irrational number, you are of course free to just try it; i.e., calculate the number to a sufficiantly large degree of digits, and run it through all known tests (there are tools for that, see above link). If the measure is good enough for your use case, and if you are aware that this is obviously useless for cryptographical applications, and always get the same numbers if you should start over, and that the quality might degrade if you get past the n you picked for testing the randomness, you could use those numbers. But it will be far better to use a dedicated (pseudo-)random number generator; and nothing beats a good physical source of randomness.






    share|cite|improve this answer









    $endgroup$








    • 4




      $begingroup$
      OK but $pi$ and $mathrme$ have the property that all the 2, 3, 4, ... digit sequences do empirically turn up with the right frequency. Nobody's managed to prove it but it seem to be true.
      $endgroup$
      – David Richerby
      Apr 10 at 16:47






    • 3




      $begingroup$
      Ayrat's answer links to other sites where mathematicians have done these tests. They believe, but haven't proved, that π meets the statistical tests.
      $endgroup$
      – Barmar
      Apr 10 at 22:27










    • $begingroup$
      Yes, that's what I meant with my last paragraph - just empirically trying it is worthwhile; but rigorously it has not been proven (or cannot simply be assumed to be true) for arbitrary "complicated-looking" irrationals. @DavidRicherby, @ Barmar
      $endgroup$
      – AnoE
      Apr 12 at 14:44


















    1












    $begingroup$

    It provides a good random number right until you realize how it was produced, as with many pseudo random number. The irrational (non algebraic and non transcendental) numbers you have chosen are common and so easier guessed then others. I can see no issue with this method provided you choose less commonly seen generators.






    share|cite|improve this answer








    New contributor




    marshal craft is a new contributor to this site. Take care in asking for clarification, commenting, and answering.
    Check out our Code of Conduct.






    $endgroup$








    • 4




      $begingroup$
      No issue except the gross inefficiency, the fact that you're relying on any adversary not knowing what your algorithm is, the fact that a bad choice of generator could lead to very poor sequence, ...
      $endgroup$
      – David Richerby
      Apr 11 at 9:16






    • 4




      $begingroup$
      By the way, of the numbers suggested in the question, $sqrt2$ is algebraic and $pi$ and $mathrme$ are transcendental.
      $endgroup$
      – David Richerby
      Apr 11 at 9:17










    • $begingroup$
      A transcendental number is a real number that is not algebraic. It is not possible for a real number to be both non algebraic and non transcendental.
      $endgroup$
      – Brady Gilg
      Apr 12 at 17:50











    Your Answer








    StackExchange.ready(function()
    var channelOptions =
    tags: "".split(" "),
    id: "419"
    ;
    initTagRenderer("".split(" "), "".split(" "), channelOptions);

    StackExchange.using("externalEditor", function()
    // Have to fire editor after snippets, if snippets enabled
    if (StackExchange.settings.snippets.snippetsEnabled)
    StackExchange.using("snippets", function()
    createEditor();
    );

    else
    createEditor();

    );

    function createEditor()
    StackExchange.prepareEditor(
    heartbeatType: 'answer',
    autoActivateHeartbeat: false,
    convertImagesToLinks: false,
    noModals: true,
    showLowRepImageUploadWarning: true,
    reputationToPostImages: null,
    bindNavPrevention: true,
    postfix: "",
    imageUploader:
    brandingHtml: "Powered by u003ca class="icon-imgur-white" href="https://imgur.com/"u003eu003c/au003e",
    contentPolicyHtml: "User contributions licensed under u003ca href="https://creativecommons.org/licenses/by-sa/3.0/"u003ecc by-sa 3.0 with attribution requiredu003c/au003e u003ca href="https://stackoverflow.com/legal/content-policy"u003e(content policy)u003c/au003e",
    allowUrls: true
    ,
    onDemand: true,
    discardSelector: ".discard-answer"
    ,immediatelyShowMarkdownHelp:true
    );



    );






    Abhradeep Sarkar is a new contributor. Be nice, and check out our Code of Conduct.









    draft saved

    draft discarded


















    StackExchange.ready(
    function ()
    StackExchange.openid.initPostLogin('.new-post-login', 'https%3a%2f%2fcs.stackexchange.com%2fquestions%2f106774%2fcan-we-generate-random-numbers-using-irrational-numbers-like-%25cf%2580-and-e%23new-answer', 'question_page');

    );

    Post as a guest















    Required, but never shown

























    6 Answers
    6






    active

    oldest

    votes








    6 Answers
    6






    active

    oldest

    votes









    active

    oldest

    votes






    active

    oldest

    votes









    13












    $begingroup$

    The most obvious disadvantage is the unnecessary complexity of PRNG algorithms based on irrational numbers. They require much more computations per generated digit than, say, an LCG; and this complexity typically grows as you go further in the sequence. Calculating 256 bits of π at the two-quadrillionth bit took 23 days on 1000 computers (back in 2010) - a rather prohibitive complexity for an RNG.






    share|cite|improve this answer









    $endgroup$

















      13












      $begingroup$

      The most obvious disadvantage is the unnecessary complexity of PRNG algorithms based on irrational numbers. They require much more computations per generated digit than, say, an LCG; and this complexity typically grows as you go further in the sequence. Calculating 256 bits of π at the two-quadrillionth bit took 23 days on 1000 computers (back in 2010) - a rather prohibitive complexity for an RNG.






      share|cite|improve this answer









      $endgroup$















        13












        13








        13





        $begingroup$

        The most obvious disadvantage is the unnecessary complexity of PRNG algorithms based on irrational numbers. They require much more computations per generated digit than, say, an LCG; and this complexity typically grows as you go further in the sequence. Calculating 256 bits of π at the two-quadrillionth bit took 23 days on 1000 computers (back in 2010) - a rather prohibitive complexity for an RNG.






        share|cite|improve this answer









        $endgroup$



        The most obvious disadvantage is the unnecessary complexity of PRNG algorithms based on irrational numbers. They require much more computations per generated digit than, say, an LCG; and this complexity typically grows as you go further in the sequence. Calculating 256 bits of π at the two-quadrillionth bit took 23 days on 1000 computers (back in 2010) - a rather prohibitive complexity for an RNG.







        share|cite|improve this answer












        share|cite|improve this answer



        share|cite|improve this answer










        answered Apr 11 at 13:37









        Dmitry GrigoryevDmitry Grigoryev

        36615




        36615





















            44












            $begingroup$

            For any reasonable definition of perfect, the mechanism you describe is not a perfect random number generator.



            • Non-repeating isn't enough. The decimal number $0.101001000100001dots$ is non-repeating but it's a terrible generator of random digits, since the answer is "always" zero, occasionally one, and never anything else.


            • We don't actually know if every digit occurs equally often in the decimal expansion of $pi$ or $mathrme$ (though we suspect they do).


            • In many situations, we require random numbers to be unpredictable (indeed, if you asked a random person what "random" means, they'd probably say something about unpredictability). The digits of well-known constants are totally predictable.


            • We usually want to generate random numbers reasonably quickly, but generating successive digits of mathematical constants tends to be quite expensive.


            • It is, however, true that the digits of $pi$ and $mathrme$ look statistically random, in the sense that every possible sequence of digits seems to occur about as often as it should. So, for example, each digit does occur very close to one time in ten; each two-digit sequence very close to one in a hundred, and so on.






            share|cite|improve this answer











            $endgroup$








            • 11




              $begingroup$
              For the third point, there must be some sort of 'secret' input to your generation process for it to be unpredictable (the generation process itself should be deterministic if we don't want to rely on yet another random number generator.). This extra input is often called a seed.
              $endgroup$
              – Discrete lizard
              Apr 10 at 14:01






            • 6




              $begingroup$
              @Discretelizard This is true but there's not much scope for seeding beyond "return successive digits starting with position $s$." By the time you've seen $2log s$ digits, that sequence occurs only a few times within the first $s^2$ digits of $pi$, so it's unique within the first $s$ digits with high probability and you know the seed.
              $endgroup$
              – David Richerby
              Apr 10 at 16:15







            • 2




              $begingroup$
              @Barmar: At that point you have to ask whether this technique is really more performant (and more space-efficient) than a "standard" PRNG would be.
              $endgroup$
              – Kevin
              Apr 11 at 0:53







            • 2




              $begingroup$
              The digits of pi or e are completely unpredictable, especially since the viewer / recipient / code breaker etc has no idea how far along in the sequence you already are. If you start at digit number 237423 of the sequence, it will take so long to figure out, as to be random.
              $endgroup$
              – DaveBoltman
              Apr 11 at 6:12






            • 10




              $begingroup$
              @DaveBoltman If we're not doing something like cryptography, nobody's going to care enough to bother figuring it out. If we are doing cryptography, it's a standard assumption that your adversary knows what algorithm you're using which, in this case, includes what irrational number the sequence is coming from and how you're choosing the digits, except for any parameter such as "start at digit $s$". If the adversary doesn't know what number you're using then, sure, the next digit could be literally anything, but then they guess it's $sqrttextmy birthday$ and the game's up.
              $endgroup$
              – David Richerby
              Apr 11 at 8:52















            44












            $begingroup$

            For any reasonable definition of perfect, the mechanism you describe is not a perfect random number generator.



            • Non-repeating isn't enough. The decimal number $0.101001000100001dots$ is non-repeating but it's a terrible generator of random digits, since the answer is "always" zero, occasionally one, and never anything else.


            • We don't actually know if every digit occurs equally often in the decimal expansion of $pi$ or $mathrme$ (though we suspect they do).


            • In many situations, we require random numbers to be unpredictable (indeed, if you asked a random person what "random" means, they'd probably say something about unpredictability). The digits of well-known constants are totally predictable.


            • We usually want to generate random numbers reasonably quickly, but generating successive digits of mathematical constants tends to be quite expensive.


            • It is, however, true that the digits of $pi$ and $mathrme$ look statistically random, in the sense that every possible sequence of digits seems to occur about as often as it should. So, for example, each digit does occur very close to one time in ten; each two-digit sequence very close to one in a hundred, and so on.






            share|cite|improve this answer











            $endgroup$








            • 11




              $begingroup$
              For the third point, there must be some sort of 'secret' input to your generation process for it to be unpredictable (the generation process itself should be deterministic if we don't want to rely on yet another random number generator.). This extra input is often called a seed.
              $endgroup$
              – Discrete lizard
              Apr 10 at 14:01






            • 6




              $begingroup$
              @Discretelizard This is true but there's not much scope for seeding beyond "return successive digits starting with position $s$." By the time you've seen $2log s$ digits, that sequence occurs only a few times within the first $s^2$ digits of $pi$, so it's unique within the first $s$ digits with high probability and you know the seed.
              $endgroup$
              – David Richerby
              Apr 10 at 16:15







            • 2




              $begingroup$
              @Barmar: At that point you have to ask whether this technique is really more performant (and more space-efficient) than a "standard" PRNG would be.
              $endgroup$
              – Kevin
              Apr 11 at 0:53







            • 2




              $begingroup$
              The digits of pi or e are completely unpredictable, especially since the viewer / recipient / code breaker etc has no idea how far along in the sequence you already are. If you start at digit number 237423 of the sequence, it will take so long to figure out, as to be random.
              $endgroup$
              – DaveBoltman
              Apr 11 at 6:12






            • 10




              $begingroup$
              @DaveBoltman If we're not doing something like cryptography, nobody's going to care enough to bother figuring it out. If we are doing cryptography, it's a standard assumption that your adversary knows what algorithm you're using which, in this case, includes what irrational number the sequence is coming from and how you're choosing the digits, except for any parameter such as "start at digit $s$". If the adversary doesn't know what number you're using then, sure, the next digit could be literally anything, but then they guess it's $sqrttextmy birthday$ and the game's up.
              $endgroup$
              – David Richerby
              Apr 11 at 8:52













            44












            44








            44





            $begingroup$

            For any reasonable definition of perfect, the mechanism you describe is not a perfect random number generator.



            • Non-repeating isn't enough. The decimal number $0.101001000100001dots$ is non-repeating but it's a terrible generator of random digits, since the answer is "always" zero, occasionally one, and never anything else.


            • We don't actually know if every digit occurs equally often in the decimal expansion of $pi$ or $mathrme$ (though we suspect they do).


            • In many situations, we require random numbers to be unpredictable (indeed, if you asked a random person what "random" means, they'd probably say something about unpredictability). The digits of well-known constants are totally predictable.


            • We usually want to generate random numbers reasonably quickly, but generating successive digits of mathematical constants tends to be quite expensive.


            • It is, however, true that the digits of $pi$ and $mathrme$ look statistically random, in the sense that every possible sequence of digits seems to occur about as often as it should. So, for example, each digit does occur very close to one time in ten; each two-digit sequence very close to one in a hundred, and so on.






            share|cite|improve this answer











            $endgroup$



            For any reasonable definition of perfect, the mechanism you describe is not a perfect random number generator.



            • Non-repeating isn't enough. The decimal number $0.101001000100001dots$ is non-repeating but it's a terrible generator of random digits, since the answer is "always" zero, occasionally one, and never anything else.


            • We don't actually know if every digit occurs equally often in the decimal expansion of $pi$ or $mathrme$ (though we suspect they do).


            • In many situations, we require random numbers to be unpredictable (indeed, if you asked a random person what "random" means, they'd probably say something about unpredictability). The digits of well-known constants are totally predictable.


            • We usually want to generate random numbers reasonably quickly, but generating successive digits of mathematical constants tends to be quite expensive.


            • It is, however, true that the digits of $pi$ and $mathrme$ look statistically random, in the sense that every possible sequence of digits seems to occur about as often as it should. So, for example, each digit does occur very close to one time in ten; each two-digit sequence very close to one in a hundred, and so on.







            share|cite|improve this answer














            share|cite|improve this answer



            share|cite|improve this answer








            edited Apr 11 at 8:30

























            answered Apr 10 at 13:36









            David RicherbyDavid Richerby

            70.4k16107196




            70.4k16107196







            • 11




              $begingroup$
              For the third point, there must be some sort of 'secret' input to your generation process for it to be unpredictable (the generation process itself should be deterministic if we don't want to rely on yet another random number generator.). This extra input is often called a seed.
              $endgroup$
              – Discrete lizard
              Apr 10 at 14:01






            • 6




              $begingroup$
              @Discretelizard This is true but there's not much scope for seeding beyond "return successive digits starting with position $s$." By the time you've seen $2log s$ digits, that sequence occurs only a few times within the first $s^2$ digits of $pi$, so it's unique within the first $s$ digits with high probability and you know the seed.
              $endgroup$
              – David Richerby
              Apr 10 at 16:15







            • 2




              $begingroup$
              @Barmar: At that point you have to ask whether this technique is really more performant (and more space-efficient) than a "standard" PRNG would be.
              $endgroup$
              – Kevin
              Apr 11 at 0:53







            • 2




              $begingroup$
              The digits of pi or e are completely unpredictable, especially since the viewer / recipient / code breaker etc has no idea how far along in the sequence you already are. If you start at digit number 237423 of the sequence, it will take so long to figure out, as to be random.
              $endgroup$
              – DaveBoltman
              Apr 11 at 6:12






            • 10




              $begingroup$
              @DaveBoltman If we're not doing something like cryptography, nobody's going to care enough to bother figuring it out. If we are doing cryptography, it's a standard assumption that your adversary knows what algorithm you're using which, in this case, includes what irrational number the sequence is coming from and how you're choosing the digits, except for any parameter such as "start at digit $s$". If the adversary doesn't know what number you're using then, sure, the next digit could be literally anything, but then they guess it's $sqrttextmy birthday$ and the game's up.
              $endgroup$
              – David Richerby
              Apr 11 at 8:52












            • 11




              $begingroup$
              For the third point, there must be some sort of 'secret' input to your generation process for it to be unpredictable (the generation process itself should be deterministic if we don't want to rely on yet another random number generator.). This extra input is often called a seed.
              $endgroup$
              – Discrete lizard
              Apr 10 at 14:01






            • 6




              $begingroup$
              @Discretelizard This is true but there's not much scope for seeding beyond "return successive digits starting with position $s$." By the time you've seen $2log s$ digits, that sequence occurs only a few times within the first $s^2$ digits of $pi$, so it's unique within the first $s$ digits with high probability and you know the seed.
              $endgroup$
              – David Richerby
              Apr 10 at 16:15







            • 2




              $begingroup$
              @Barmar: At that point you have to ask whether this technique is really more performant (and more space-efficient) than a "standard" PRNG would be.
              $endgroup$
              – Kevin
              Apr 11 at 0:53







            • 2




              $begingroup$
              The digits of pi or e are completely unpredictable, especially since the viewer / recipient / code breaker etc has no idea how far along in the sequence you already are. If you start at digit number 237423 of the sequence, it will take so long to figure out, as to be random.
              $endgroup$
              – DaveBoltman
              Apr 11 at 6:12






            • 10




              $begingroup$
              @DaveBoltman If we're not doing something like cryptography, nobody's going to care enough to bother figuring it out. If we are doing cryptography, it's a standard assumption that your adversary knows what algorithm you're using which, in this case, includes what irrational number the sequence is coming from and how you're choosing the digits, except for any parameter such as "start at digit $s$". If the adversary doesn't know what number you're using then, sure, the next digit could be literally anything, but then they guess it's $sqrttextmy birthday$ and the game's up.
              $endgroup$
              – David Richerby
              Apr 11 at 8:52







            11




            11




            $begingroup$
            For the third point, there must be some sort of 'secret' input to your generation process for it to be unpredictable (the generation process itself should be deterministic if we don't want to rely on yet another random number generator.). This extra input is often called a seed.
            $endgroup$
            – Discrete lizard
            Apr 10 at 14:01




            $begingroup$
            For the third point, there must be some sort of 'secret' input to your generation process for it to be unpredictable (the generation process itself should be deterministic if we don't want to rely on yet another random number generator.). This extra input is often called a seed.
            $endgroup$
            – Discrete lizard
            Apr 10 at 14:01




            6




            6




            $begingroup$
            @Discretelizard This is true but there's not much scope for seeding beyond "return successive digits starting with position $s$." By the time you've seen $2log s$ digits, that sequence occurs only a few times within the first $s^2$ digits of $pi$, so it's unique within the first $s$ digits with high probability and you know the seed.
            $endgroup$
            – David Richerby
            Apr 10 at 16:15





            $begingroup$
            @Discretelizard This is true but there's not much scope for seeding beyond "return successive digits starting with position $s$." By the time you've seen $2log s$ digits, that sequence occurs only a few times within the first $s^2$ digits of $pi$, so it's unique within the first $s$ digits with high probability and you know the seed.
            $endgroup$
            – David Richerby
            Apr 10 at 16:15





            2




            2




            $begingroup$
            @Barmar: At that point you have to ask whether this technique is really more performant (and more space-efficient) than a "standard" PRNG would be.
            $endgroup$
            – Kevin
            Apr 11 at 0:53





            $begingroup$
            @Barmar: At that point you have to ask whether this technique is really more performant (and more space-efficient) than a "standard" PRNG would be.
            $endgroup$
            – Kevin
            Apr 11 at 0:53





            2




            2




            $begingroup$
            The digits of pi or e are completely unpredictable, especially since the viewer / recipient / code breaker etc has no idea how far along in the sequence you already are. If you start at digit number 237423 of the sequence, it will take so long to figure out, as to be random.
            $endgroup$
            – DaveBoltman
            Apr 11 at 6:12




            $begingroup$
            The digits of pi or e are completely unpredictable, especially since the viewer / recipient / code breaker etc has no idea how far along in the sequence you already are. If you start at digit number 237423 of the sequence, it will take so long to figure out, as to be random.
            $endgroup$
            – DaveBoltman
            Apr 11 at 6:12




            10




            10




            $begingroup$
            @DaveBoltman If we're not doing something like cryptography, nobody's going to care enough to bother figuring it out. If we are doing cryptography, it's a standard assumption that your adversary knows what algorithm you're using which, in this case, includes what irrational number the sequence is coming from and how you're choosing the digits, except for any parameter such as "start at digit $s$". If the adversary doesn't know what number you're using then, sure, the next digit could be literally anything, but then they guess it's $sqrttextmy birthday$ and the game's up.
            $endgroup$
            – David Richerby
            Apr 11 at 8:52




            $begingroup$
            @DaveBoltman If we're not doing something like cryptography, nobody's going to care enough to bother figuring it out. If we are doing cryptography, it's a standard assumption that your adversary knows what algorithm you're using which, in this case, includes what irrational number the sequence is coming from and how you're choosing the digits, except for any parameter such as "start at digit $s$". If the adversary doesn't know what number you're using then, sure, the next digit could be literally anything, but then they guess it's $sqrttextmy birthday$ and the game's up.
            $endgroup$
            – David Richerby
            Apr 11 at 8:52











            28












            $begingroup$

            It is cryptographically useless because an adversary can predict every single digit. It is also very time consuming.






            share|cite|improve this answer









            $endgroup$








            • 9




              $begingroup$
              OP never mentions cryptography...
              $endgroup$
              – AnoE
              Apr 10 at 16:19






            • 13




              $begingroup$
              @AnoE So? That this process would be cryptographically useless is still relevant because crypto is an avid user of randomness. If you bring up the devices /dev/random and /dev/urandom someone will invariably bring up cryptography.
              $endgroup$
              – Greg Schmit
              Apr 10 at 16:31






            • 6




              $begingroup$
              You would be amazed at how useless cryptographic security is in real time PRNG generation. irrational numbers are often used in GPU PRNGs. There are a lot of applications where how "secure" your PRNG is simply irrelevant. What matters in something like coherent noise generation is the quality of distribution and how often your period repeats, and correlation effects due to adjacent seeds (which would require avalanche mixers to fix). Quite honestly your answer is wrong, doesn't belong here, and should probably be deleted.
              $endgroup$
              – opa
              Apr 10 at 17:44







            • 6




              $begingroup$
              This is anot an answer to the question. Note the OP of the linked question uses random numbers for seeding a monte carlo analysis. An update to address the question asked should be considered. mathoverflow.net/questions/26942/…
              $endgroup$
              – CramerTV
              Apr 10 at 17:47







            • 8




              $begingroup$
              Certainly there are many applications where PRNGs don't need to be cryptographically secure. But OP didn't ask if it was useful for some purposes, they asked if this method was a "perfect RNG". While they haven't clarified what they mean by "perfect", the fact that it's unsuitable for one of the major uses of RNGs seems very relevant to answering that question.
              $endgroup$
              – Geoffrey Brent
              Apr 11 at 2:08
















            28












            $begingroup$

            It is cryptographically useless because an adversary can predict every single digit. It is also very time consuming.






            share|cite|improve this answer









            $endgroup$








            • 9




              $begingroup$
              OP never mentions cryptography...
              $endgroup$
              – AnoE
              Apr 10 at 16:19






            • 13




              $begingroup$
              @AnoE So? That this process would be cryptographically useless is still relevant because crypto is an avid user of randomness. If you bring up the devices /dev/random and /dev/urandom someone will invariably bring up cryptography.
              $endgroup$
              – Greg Schmit
              Apr 10 at 16:31






            • 6




              $begingroup$
              You would be amazed at how useless cryptographic security is in real time PRNG generation. irrational numbers are often used in GPU PRNGs. There are a lot of applications where how "secure" your PRNG is simply irrelevant. What matters in something like coherent noise generation is the quality of distribution and how often your period repeats, and correlation effects due to adjacent seeds (which would require avalanche mixers to fix). Quite honestly your answer is wrong, doesn't belong here, and should probably be deleted.
              $endgroup$
              – opa
              Apr 10 at 17:44







            • 6




              $begingroup$
              This is anot an answer to the question. Note the OP of the linked question uses random numbers for seeding a monte carlo analysis. An update to address the question asked should be considered. mathoverflow.net/questions/26942/…
              $endgroup$
              – CramerTV
              Apr 10 at 17:47







            • 8




              $begingroup$
              Certainly there are many applications where PRNGs don't need to be cryptographically secure. But OP didn't ask if it was useful for some purposes, they asked if this method was a "perfect RNG". While they haven't clarified what they mean by "perfect", the fact that it's unsuitable for one of the major uses of RNGs seems very relevant to answering that question.
              $endgroup$
              – Geoffrey Brent
              Apr 11 at 2:08














            28












            28








            28





            $begingroup$

            It is cryptographically useless because an adversary can predict every single digit. It is also very time consuming.






            share|cite|improve this answer









            $endgroup$



            It is cryptographically useless because an adversary can predict every single digit. It is also very time consuming.







            share|cite|improve this answer












            share|cite|improve this answer



            share|cite|improve this answer










            answered Apr 10 at 13:01









            gnasher729gnasher729

            12k1318




            12k1318







            • 9




              $begingroup$
              OP never mentions cryptography...
              $endgroup$
              – AnoE
              Apr 10 at 16:19






            • 13




              $begingroup$
              @AnoE So? That this process would be cryptographically useless is still relevant because crypto is an avid user of randomness. If you bring up the devices /dev/random and /dev/urandom someone will invariably bring up cryptography.
              $endgroup$
              – Greg Schmit
              Apr 10 at 16:31






            • 6




              $begingroup$
              You would be amazed at how useless cryptographic security is in real time PRNG generation. irrational numbers are often used in GPU PRNGs. There are a lot of applications where how "secure" your PRNG is simply irrelevant. What matters in something like coherent noise generation is the quality of distribution and how often your period repeats, and correlation effects due to adjacent seeds (which would require avalanche mixers to fix). Quite honestly your answer is wrong, doesn't belong here, and should probably be deleted.
              $endgroup$
              – opa
              Apr 10 at 17:44







            • 6




              $begingroup$
              This is anot an answer to the question. Note the OP of the linked question uses random numbers for seeding a monte carlo analysis. An update to address the question asked should be considered. mathoverflow.net/questions/26942/…
              $endgroup$
              – CramerTV
              Apr 10 at 17:47







            • 8




              $begingroup$
              Certainly there are many applications where PRNGs don't need to be cryptographically secure. But OP didn't ask if it was useful for some purposes, they asked if this method was a "perfect RNG". While they haven't clarified what they mean by "perfect", the fact that it's unsuitable for one of the major uses of RNGs seems very relevant to answering that question.
              $endgroup$
              – Geoffrey Brent
              Apr 11 at 2:08













            • 9




              $begingroup$
              OP never mentions cryptography...
              $endgroup$
              – AnoE
              Apr 10 at 16:19






            • 13




              $begingroup$
              @AnoE So? That this process would be cryptographically useless is still relevant because crypto is an avid user of randomness. If you bring up the devices /dev/random and /dev/urandom someone will invariably bring up cryptography.
              $endgroup$
              – Greg Schmit
              Apr 10 at 16:31






            • 6




              $begingroup$
              You would be amazed at how useless cryptographic security is in real time PRNG generation. irrational numbers are often used in GPU PRNGs. There are a lot of applications where how "secure" your PRNG is simply irrelevant. What matters in something like coherent noise generation is the quality of distribution and how often your period repeats, and correlation effects due to adjacent seeds (which would require avalanche mixers to fix). Quite honestly your answer is wrong, doesn't belong here, and should probably be deleted.
              $endgroup$
              – opa
              Apr 10 at 17:44







            • 6




              $begingroup$
              This is anot an answer to the question. Note the OP of the linked question uses random numbers for seeding a monte carlo analysis. An update to address the question asked should be considered. mathoverflow.net/questions/26942/…
              $endgroup$
              – CramerTV
              Apr 10 at 17:47







            • 8




              $begingroup$
              Certainly there are many applications where PRNGs don't need to be cryptographically secure. But OP didn't ask if it was useful for some purposes, they asked if this method was a "perfect RNG". While they haven't clarified what they mean by "perfect", the fact that it's unsuitable for one of the major uses of RNGs seems very relevant to answering that question.
              $endgroup$
              – Geoffrey Brent
              Apr 11 at 2:08








            9




            9




            $begingroup$
            OP never mentions cryptography...
            $endgroup$
            – AnoE
            Apr 10 at 16:19




            $begingroup$
            OP never mentions cryptography...
            $endgroup$
            – AnoE
            Apr 10 at 16:19




            13




            13




            $begingroup$
            @AnoE So? That this process would be cryptographically useless is still relevant because crypto is an avid user of randomness. If you bring up the devices /dev/random and /dev/urandom someone will invariably bring up cryptography.
            $endgroup$
            – Greg Schmit
            Apr 10 at 16:31




            $begingroup$
            @AnoE So? That this process would be cryptographically useless is still relevant because crypto is an avid user of randomness. If you bring up the devices /dev/random and /dev/urandom someone will invariably bring up cryptography.
            $endgroup$
            – Greg Schmit
            Apr 10 at 16:31




            6




            6




            $begingroup$
            You would be amazed at how useless cryptographic security is in real time PRNG generation. irrational numbers are often used in GPU PRNGs. There are a lot of applications where how "secure" your PRNG is simply irrelevant. What matters in something like coherent noise generation is the quality of distribution and how often your period repeats, and correlation effects due to adjacent seeds (which would require avalanche mixers to fix). Quite honestly your answer is wrong, doesn't belong here, and should probably be deleted.
            $endgroup$
            – opa
            Apr 10 at 17:44





            $begingroup$
            You would be amazed at how useless cryptographic security is in real time PRNG generation. irrational numbers are often used in GPU PRNGs. There are a lot of applications where how "secure" your PRNG is simply irrelevant. What matters in something like coherent noise generation is the quality of distribution and how often your period repeats, and correlation effects due to adjacent seeds (which would require avalanche mixers to fix). Quite honestly your answer is wrong, doesn't belong here, and should probably be deleted.
            $endgroup$
            – opa
            Apr 10 at 17:44





            6




            6




            $begingroup$
            This is anot an answer to the question. Note the OP of the linked question uses random numbers for seeding a monte carlo analysis. An update to address the question asked should be considered. mathoverflow.net/questions/26942/…
            $endgroup$
            – CramerTV
            Apr 10 at 17:47





            $begingroup$
            This is anot an answer to the question. Note the OP of the linked question uses random numbers for seeding a monte carlo analysis. An update to address the question asked should be considered. mathoverflow.net/questions/26942/…
            $endgroup$
            – CramerTV
            Apr 10 at 17:47





            8




            8




            $begingroup$
            Certainly there are many applications where PRNGs don't need to be cryptographically secure. But OP didn't ask if it was useful for some purposes, they asked if this method was a "perfect RNG". While they haven't clarified what they mean by "perfect", the fact that it's unsuitable for one of the major uses of RNGs seems very relevant to answering that question.
            $endgroup$
            – Geoffrey Brent
            Apr 11 at 2:08





            $begingroup$
            Certainly there are many applications where PRNGs don't need to be cryptographically secure. But OP didn't ask if it was useful for some purposes, they asked if this method was a "perfect RNG". While they haven't clarified what they mean by "perfect", the fact that it's unsuitable for one of the major uses of RNGs seems very relevant to answering that question.
            $endgroup$
            – Geoffrey Brent
            Apr 11 at 2:08












            6












            $begingroup$

            (updated after many people pointed out that random number generator is not the same thing as a single normal sequence)



            If you ask whether you can get a normal sequence out of $pi$ (i.e., all numbers appear uniformly), then there are several answers on mathoverflow. For example, the answer about Distribution of the digits of Pi says:




            ...it is believed that $pi$ is a normal number (~uniform distribution of every digits sequence).




            For digit distribution data, see e.g. http://www.eveandersson.com/pi/precalculated-frequencies or https://thestarman.pcministry.com/math/pi/RandPI.html (first 1000 digits):



            enter image description here



            At mathoverflow, there are also nice answers at:



            • What is the state of our ignorance about the normality of pi?

            • Does pi contain 1000 consecutive zeroes?





            share|cite|improve this answer











            $endgroup$








            • 3




              $begingroup$
              If you believe the question is a duplicate, then why are you answering it? You should simply flag it, not reinforce undesired posting behavior.
              $endgroup$
              – dkaeae
              Apr 10 at 16:11






            • 8




              $begingroup$
              @dkaeae There is no support for duplicates of questions on other sites. Furthermore, the same question on different sites can get different answers. In this case, a site such as Mathematics might not give much consideration to security concerns. See also this answer. Do note that we discourage asking the same question on multiple sites at the same time, since this tends to lead to wasted efforts. But the same question by different persons at different times on different sites is usually ok.
              $endgroup$
              – Discrete lizard
              Apr 10 at 17:01






            • 6




              $begingroup$
              Unfortunately, just because a number is normal doesn't mean that outputting its digits gives you a good RNG. The outputs of such a RNG are still entirely predictable. Whether that's acceptable might depend on the application. So, I don't think it's quite as simple as saying "pi is normal, case closed".
              $endgroup$
              – D.W.
              Apr 10 at 18:06







            • 2




              $begingroup$
              That is just emperical observation for first few digits? What is to be meant by that?
              $endgroup$
              – marshal craft
              Apr 11 at 6:23






            • 1




              $begingroup$
              @D.W. I mentioned that I intend to use a combination of numbers like π and e. And please say how the output will be predictable if we do not know how far down the sequence the generator has gone?
              $endgroup$
              – Abhradeep Sarkar
              Apr 11 at 8:19















            6












            $begingroup$

            (updated after many people pointed out that random number generator is not the same thing as a single normal sequence)



            If you ask whether you can get a normal sequence out of $pi$ (i.e., all numbers appear uniformly), then there are several answers on mathoverflow. For example, the answer about Distribution of the digits of Pi says:




            ...it is believed that $pi$ is a normal number (~uniform distribution of every digits sequence).




            For digit distribution data, see e.g. http://www.eveandersson.com/pi/precalculated-frequencies or https://thestarman.pcministry.com/math/pi/RandPI.html (first 1000 digits):



            enter image description here



            At mathoverflow, there are also nice answers at:



            • What is the state of our ignorance about the normality of pi?

            • Does pi contain 1000 consecutive zeroes?





            share|cite|improve this answer











            $endgroup$








            • 3




              $begingroup$
              If you believe the question is a duplicate, then why are you answering it? You should simply flag it, not reinforce undesired posting behavior.
              $endgroup$
              – dkaeae
              Apr 10 at 16:11






            • 8




              $begingroup$
              @dkaeae There is no support for duplicates of questions on other sites. Furthermore, the same question on different sites can get different answers. In this case, a site such as Mathematics might not give much consideration to security concerns. See also this answer. Do note that we discourage asking the same question on multiple sites at the same time, since this tends to lead to wasted efforts. But the same question by different persons at different times on different sites is usually ok.
              $endgroup$
              – Discrete lizard
              Apr 10 at 17:01






            • 6




              $begingroup$
              Unfortunately, just because a number is normal doesn't mean that outputting its digits gives you a good RNG. The outputs of such a RNG are still entirely predictable. Whether that's acceptable might depend on the application. So, I don't think it's quite as simple as saying "pi is normal, case closed".
              $endgroup$
              – D.W.
              Apr 10 at 18:06







            • 2




              $begingroup$
              That is just emperical observation for first few digits? What is to be meant by that?
              $endgroup$
              – marshal craft
              Apr 11 at 6:23






            • 1




              $begingroup$
              @D.W. I mentioned that I intend to use a combination of numbers like π and e. And please say how the output will be predictable if we do not know how far down the sequence the generator has gone?
              $endgroup$
              – Abhradeep Sarkar
              Apr 11 at 8:19













            6












            6








            6





            $begingroup$

            (updated after many people pointed out that random number generator is not the same thing as a single normal sequence)



            If you ask whether you can get a normal sequence out of $pi$ (i.e., all numbers appear uniformly), then there are several answers on mathoverflow. For example, the answer about Distribution of the digits of Pi says:




            ...it is believed that $pi$ is a normal number (~uniform distribution of every digits sequence).




            For digit distribution data, see e.g. http://www.eveandersson.com/pi/precalculated-frequencies or https://thestarman.pcministry.com/math/pi/RandPI.html (first 1000 digits):



            enter image description here



            At mathoverflow, there are also nice answers at:



            • What is the state of our ignorance about the normality of pi?

            • Does pi contain 1000 consecutive zeroes?





            share|cite|improve this answer











            $endgroup$



            (updated after many people pointed out that random number generator is not the same thing as a single normal sequence)



            If you ask whether you can get a normal sequence out of $pi$ (i.e., all numbers appear uniformly), then there are several answers on mathoverflow. For example, the answer about Distribution of the digits of Pi says:




            ...it is believed that $pi$ is a normal number (~uniform distribution of every digits sequence).




            For digit distribution data, see e.g. http://www.eveandersson.com/pi/precalculated-frequencies or https://thestarman.pcministry.com/math/pi/RandPI.html (first 1000 digits):



            enter image description here



            At mathoverflow, there are also nice answers at:



            • What is the state of our ignorance about the normality of pi?

            • Does pi contain 1000 consecutive zeroes?






            share|cite|improve this answer














            share|cite|improve this answer



            share|cite|improve this answer








            edited Apr 12 at 12:07

























            answered Apr 10 at 13:33









            AyratAyrat

            5191420




            5191420







            • 3




              $begingroup$
              If you believe the question is a duplicate, then why are you answering it? You should simply flag it, not reinforce undesired posting behavior.
              $endgroup$
              – dkaeae
              Apr 10 at 16:11






            • 8




              $begingroup$
              @dkaeae There is no support for duplicates of questions on other sites. Furthermore, the same question on different sites can get different answers. In this case, a site such as Mathematics might not give much consideration to security concerns. See also this answer. Do note that we discourage asking the same question on multiple sites at the same time, since this tends to lead to wasted efforts. But the same question by different persons at different times on different sites is usually ok.
              $endgroup$
              – Discrete lizard
              Apr 10 at 17:01






            • 6




              $begingroup$
              Unfortunately, just because a number is normal doesn't mean that outputting its digits gives you a good RNG. The outputs of such a RNG are still entirely predictable. Whether that's acceptable might depend on the application. So, I don't think it's quite as simple as saying "pi is normal, case closed".
              $endgroup$
              – D.W.
              Apr 10 at 18:06







            • 2




              $begingroup$
              That is just emperical observation for first few digits? What is to be meant by that?
              $endgroup$
              – marshal craft
              Apr 11 at 6:23






            • 1




              $begingroup$
              @D.W. I mentioned that I intend to use a combination of numbers like π and e. And please say how the output will be predictable if we do not know how far down the sequence the generator has gone?
              $endgroup$
              – Abhradeep Sarkar
              Apr 11 at 8:19












            • 3




              $begingroup$
              If you believe the question is a duplicate, then why are you answering it? You should simply flag it, not reinforce undesired posting behavior.
              $endgroup$
              – dkaeae
              Apr 10 at 16:11






            • 8




              $begingroup$
              @dkaeae There is no support for duplicates of questions on other sites. Furthermore, the same question on different sites can get different answers. In this case, a site such as Mathematics might not give much consideration to security concerns. See also this answer. Do note that we discourage asking the same question on multiple sites at the same time, since this tends to lead to wasted efforts. But the same question by different persons at different times on different sites is usually ok.
              $endgroup$
              – Discrete lizard
              Apr 10 at 17:01






            • 6




              $begingroup$
              Unfortunately, just because a number is normal doesn't mean that outputting its digits gives you a good RNG. The outputs of such a RNG are still entirely predictable. Whether that's acceptable might depend on the application. So, I don't think it's quite as simple as saying "pi is normal, case closed".
              $endgroup$
              – D.W.
              Apr 10 at 18:06







            • 2




              $begingroup$
              That is just emperical observation for first few digits? What is to be meant by that?
              $endgroup$
              – marshal craft
              Apr 11 at 6:23






            • 1




              $begingroup$
              @D.W. I mentioned that I intend to use a combination of numbers like π and e. And please say how the output will be predictable if we do not know how far down the sequence the generator has gone?
              $endgroup$
              – Abhradeep Sarkar
              Apr 11 at 8:19







            3




            3




            $begingroup$
            If you believe the question is a duplicate, then why are you answering it? You should simply flag it, not reinforce undesired posting behavior.
            $endgroup$
            – dkaeae
            Apr 10 at 16:11




            $begingroup$
            If you believe the question is a duplicate, then why are you answering it? You should simply flag it, not reinforce undesired posting behavior.
            $endgroup$
            – dkaeae
            Apr 10 at 16:11




            8




            8




            $begingroup$
            @dkaeae There is no support for duplicates of questions on other sites. Furthermore, the same question on different sites can get different answers. In this case, a site such as Mathematics might not give much consideration to security concerns. See also this answer. Do note that we discourage asking the same question on multiple sites at the same time, since this tends to lead to wasted efforts. But the same question by different persons at different times on different sites is usually ok.
            $endgroup$
            – Discrete lizard
            Apr 10 at 17:01




            $begingroup$
            @dkaeae There is no support for duplicates of questions on other sites. Furthermore, the same question on different sites can get different answers. In this case, a site such as Mathematics might not give much consideration to security concerns. See also this answer. Do note that we discourage asking the same question on multiple sites at the same time, since this tends to lead to wasted efforts. But the same question by different persons at different times on different sites is usually ok.
            $endgroup$
            – Discrete lizard
            Apr 10 at 17:01




            6




            6




            $begingroup$
            Unfortunately, just because a number is normal doesn't mean that outputting its digits gives you a good RNG. The outputs of such a RNG are still entirely predictable. Whether that's acceptable might depend on the application. So, I don't think it's quite as simple as saying "pi is normal, case closed".
            $endgroup$
            – D.W.
            Apr 10 at 18:06





            $begingroup$
            Unfortunately, just because a number is normal doesn't mean that outputting its digits gives you a good RNG. The outputs of such a RNG are still entirely predictable. Whether that's acceptable might depend on the application. So, I don't think it's quite as simple as saying "pi is normal, case closed".
            $endgroup$
            – D.W.
            Apr 10 at 18:06





            2




            2




            $begingroup$
            That is just emperical observation for first few digits? What is to be meant by that?
            $endgroup$
            – marshal craft
            Apr 11 at 6:23




            $begingroup$
            That is just emperical observation for first few digits? What is to be meant by that?
            $endgroup$
            – marshal craft
            Apr 11 at 6:23




            1




            1




            $begingroup$
            @D.W. I mentioned that I intend to use a combination of numbers like π and e. And please say how the output will be predictable if we do not know how far down the sequence the generator has gone?
            $endgroup$
            – Abhradeep Sarkar
            Apr 11 at 8:19




            $begingroup$
            @D.W. I mentioned that I intend to use a combination of numbers like π and e. And please say how the output will be predictable if we do not know how far down the sequence the generator has gone?
            $endgroup$
            – Abhradeep Sarkar
            Apr 11 at 8:19











            1












            $begingroup$

            In general, this approach does not work: "randomness" does not mean that you get a lot of different digits, but there are other aspects as well. For example, a classic test is to see if all two-digit, or three-digit etc. combinations occur with the same frequency. This would be a very simple test, which can rule out obvious non-random results, but is still by far too simplistic to check for really random behaviour.



            See the Wikipedia page about Randomness Tests as a collection of links to primary sources regarding this. They do mention a good amount of quite complicated-sounding concepts; it is it not so important to go into deep detail about this - but it is clear that it is not intuitively possible to declare a specific number to be a good source for such digits.



            On a positive note: For a specific irrational number, you are of course free to just try it; i.e., calculate the number to a sufficiantly large degree of digits, and run it through all known tests (there are tools for that, see above link). If the measure is good enough for your use case, and if you are aware that this is obviously useless for cryptographical applications, and always get the same numbers if you should start over, and that the quality might degrade if you get past the n you picked for testing the randomness, you could use those numbers. But it will be far better to use a dedicated (pseudo-)random number generator; and nothing beats a good physical source of randomness.






            share|cite|improve this answer









            $endgroup$








            • 4




              $begingroup$
              OK but $pi$ and $mathrme$ have the property that all the 2, 3, 4, ... digit sequences do empirically turn up with the right frequency. Nobody's managed to prove it but it seem to be true.
              $endgroup$
              – David Richerby
              Apr 10 at 16:47






            • 3




              $begingroup$
              Ayrat's answer links to other sites where mathematicians have done these tests. They believe, but haven't proved, that π meets the statistical tests.
              $endgroup$
              – Barmar
              Apr 10 at 22:27










            • $begingroup$
              Yes, that's what I meant with my last paragraph - just empirically trying it is worthwhile; but rigorously it has not been proven (or cannot simply be assumed to be true) for arbitrary "complicated-looking" irrationals. @DavidRicherby, @ Barmar
              $endgroup$
              – AnoE
              Apr 12 at 14:44















            1












            $begingroup$

            In general, this approach does not work: "randomness" does not mean that you get a lot of different digits, but there are other aspects as well. For example, a classic test is to see if all two-digit, or three-digit etc. combinations occur with the same frequency. This would be a very simple test, which can rule out obvious non-random results, but is still by far too simplistic to check for really random behaviour.



            See the Wikipedia page about Randomness Tests as a collection of links to primary sources regarding this. They do mention a good amount of quite complicated-sounding concepts; it is it not so important to go into deep detail about this - but it is clear that it is not intuitively possible to declare a specific number to be a good source for such digits.



            On a positive note: For a specific irrational number, you are of course free to just try it; i.e., calculate the number to a sufficiantly large degree of digits, and run it through all known tests (there are tools for that, see above link). If the measure is good enough for your use case, and if you are aware that this is obviously useless for cryptographical applications, and always get the same numbers if you should start over, and that the quality might degrade if you get past the n you picked for testing the randomness, you could use those numbers. But it will be far better to use a dedicated (pseudo-)random number generator; and nothing beats a good physical source of randomness.






            share|cite|improve this answer









            $endgroup$








            • 4




              $begingroup$
              OK but $pi$ and $mathrme$ have the property that all the 2, 3, 4, ... digit sequences do empirically turn up with the right frequency. Nobody's managed to prove it but it seem to be true.
              $endgroup$
              – David Richerby
              Apr 10 at 16:47






            • 3




              $begingroup$
              Ayrat's answer links to other sites where mathematicians have done these tests. They believe, but haven't proved, that π meets the statistical tests.
              $endgroup$
              – Barmar
              Apr 10 at 22:27










            • $begingroup$
              Yes, that's what I meant with my last paragraph - just empirically trying it is worthwhile; but rigorously it has not been proven (or cannot simply be assumed to be true) for arbitrary "complicated-looking" irrationals. @DavidRicherby, @ Barmar
              $endgroup$
              – AnoE
              Apr 12 at 14:44













            1












            1








            1





            $begingroup$

            In general, this approach does not work: "randomness" does not mean that you get a lot of different digits, but there are other aspects as well. For example, a classic test is to see if all two-digit, or three-digit etc. combinations occur with the same frequency. This would be a very simple test, which can rule out obvious non-random results, but is still by far too simplistic to check for really random behaviour.



            See the Wikipedia page about Randomness Tests as a collection of links to primary sources regarding this. They do mention a good amount of quite complicated-sounding concepts; it is it not so important to go into deep detail about this - but it is clear that it is not intuitively possible to declare a specific number to be a good source for such digits.



            On a positive note: For a specific irrational number, you are of course free to just try it; i.e., calculate the number to a sufficiantly large degree of digits, and run it through all known tests (there are tools for that, see above link). If the measure is good enough for your use case, and if you are aware that this is obviously useless for cryptographical applications, and always get the same numbers if you should start over, and that the quality might degrade if you get past the n you picked for testing the randomness, you could use those numbers. But it will be far better to use a dedicated (pseudo-)random number generator; and nothing beats a good physical source of randomness.






            share|cite|improve this answer









            $endgroup$



            In general, this approach does not work: "randomness" does not mean that you get a lot of different digits, but there are other aspects as well. For example, a classic test is to see if all two-digit, or three-digit etc. combinations occur with the same frequency. This would be a very simple test, which can rule out obvious non-random results, but is still by far too simplistic to check for really random behaviour.



            See the Wikipedia page about Randomness Tests as a collection of links to primary sources regarding this. They do mention a good amount of quite complicated-sounding concepts; it is it not so important to go into deep detail about this - but it is clear that it is not intuitively possible to declare a specific number to be a good source for such digits.



            On a positive note: For a specific irrational number, you are of course free to just try it; i.e., calculate the number to a sufficiantly large degree of digits, and run it through all known tests (there are tools for that, see above link). If the measure is good enough for your use case, and if you are aware that this is obviously useless for cryptographical applications, and always get the same numbers if you should start over, and that the quality might degrade if you get past the n you picked for testing the randomness, you could use those numbers. But it will be far better to use a dedicated (pseudo-)random number generator; and nothing beats a good physical source of randomness.







            share|cite|improve this answer












            share|cite|improve this answer



            share|cite|improve this answer










            answered Apr 10 at 16:29









            AnoEAnoE

            1,03749




            1,03749







            • 4




              $begingroup$
              OK but $pi$ and $mathrme$ have the property that all the 2, 3, 4, ... digit sequences do empirically turn up with the right frequency. Nobody's managed to prove it but it seem to be true.
              $endgroup$
              – David Richerby
              Apr 10 at 16:47






            • 3




              $begingroup$
              Ayrat's answer links to other sites where mathematicians have done these tests. They believe, but haven't proved, that π meets the statistical tests.
              $endgroup$
              – Barmar
              Apr 10 at 22:27










            • $begingroup$
              Yes, that's what I meant with my last paragraph - just empirically trying it is worthwhile; but rigorously it has not been proven (or cannot simply be assumed to be true) for arbitrary "complicated-looking" irrationals. @DavidRicherby, @ Barmar
              $endgroup$
              – AnoE
              Apr 12 at 14:44












            • 4




              $begingroup$
              OK but $pi$ and $mathrme$ have the property that all the 2, 3, 4, ... digit sequences do empirically turn up with the right frequency. Nobody's managed to prove it but it seem to be true.
              $endgroup$
              – David Richerby
              Apr 10 at 16:47






            • 3




              $begingroup$
              Ayrat's answer links to other sites where mathematicians have done these tests. They believe, but haven't proved, that π meets the statistical tests.
              $endgroup$
              – Barmar
              Apr 10 at 22:27










            • $begingroup$
              Yes, that's what I meant with my last paragraph - just empirically trying it is worthwhile; but rigorously it has not been proven (or cannot simply be assumed to be true) for arbitrary "complicated-looking" irrationals. @DavidRicherby, @ Barmar
              $endgroup$
              – AnoE
              Apr 12 at 14:44







            4




            4




            $begingroup$
            OK but $pi$ and $mathrme$ have the property that all the 2, 3, 4, ... digit sequences do empirically turn up with the right frequency. Nobody's managed to prove it but it seem to be true.
            $endgroup$
            – David Richerby
            Apr 10 at 16:47




            $begingroup$
            OK but $pi$ and $mathrme$ have the property that all the 2, 3, 4, ... digit sequences do empirically turn up with the right frequency. Nobody's managed to prove it but it seem to be true.
            $endgroup$
            – David Richerby
            Apr 10 at 16:47




            3




            3




            $begingroup$
            Ayrat's answer links to other sites where mathematicians have done these tests. They believe, but haven't proved, that π meets the statistical tests.
            $endgroup$
            – Barmar
            Apr 10 at 22:27




            $begingroup$
            Ayrat's answer links to other sites where mathematicians have done these tests. They believe, but haven't proved, that π meets the statistical tests.
            $endgroup$
            – Barmar
            Apr 10 at 22:27












            $begingroup$
            Yes, that's what I meant with my last paragraph - just empirically trying it is worthwhile; but rigorously it has not been proven (or cannot simply be assumed to be true) for arbitrary "complicated-looking" irrationals. @DavidRicherby, @ Barmar
            $endgroup$
            – AnoE
            Apr 12 at 14:44




            $begingroup$
            Yes, that's what I meant with my last paragraph - just empirically trying it is worthwhile; but rigorously it has not been proven (or cannot simply be assumed to be true) for arbitrary "complicated-looking" irrationals. @DavidRicherby, @ Barmar
            $endgroup$
            – AnoE
            Apr 12 at 14:44











            1












            $begingroup$

            It provides a good random number right until you realize how it was produced, as with many pseudo random number. The irrational (non algebraic and non transcendental) numbers you have chosen are common and so easier guessed then others. I can see no issue with this method provided you choose less commonly seen generators.






            share|cite|improve this answer








            New contributor




            marshal craft is a new contributor to this site. Take care in asking for clarification, commenting, and answering.
            Check out our Code of Conduct.






            $endgroup$








            • 4




              $begingroup$
              No issue except the gross inefficiency, the fact that you're relying on any adversary not knowing what your algorithm is, the fact that a bad choice of generator could lead to very poor sequence, ...
              $endgroup$
              – David Richerby
              Apr 11 at 9:16






            • 4




              $begingroup$
              By the way, of the numbers suggested in the question, $sqrt2$ is algebraic and $pi$ and $mathrme$ are transcendental.
              $endgroup$
              – David Richerby
              Apr 11 at 9:17










            • $begingroup$
              A transcendental number is a real number that is not algebraic. It is not possible for a real number to be both non algebraic and non transcendental.
              $endgroup$
              – Brady Gilg
              Apr 12 at 17:50















            1












            $begingroup$

            It provides a good random number right until you realize how it was produced, as with many pseudo random number. The irrational (non algebraic and non transcendental) numbers you have chosen are common and so easier guessed then others. I can see no issue with this method provided you choose less commonly seen generators.






            share|cite|improve this answer








            New contributor




            marshal craft is a new contributor to this site. Take care in asking for clarification, commenting, and answering.
            Check out our Code of Conduct.






            $endgroup$








            • 4




              $begingroup$
              No issue except the gross inefficiency, the fact that you're relying on any adversary not knowing what your algorithm is, the fact that a bad choice of generator could lead to very poor sequence, ...
              $endgroup$
              – David Richerby
              Apr 11 at 9:16






            • 4




              $begingroup$
              By the way, of the numbers suggested in the question, $sqrt2$ is algebraic and $pi$ and $mathrme$ are transcendental.
              $endgroup$
              – David Richerby
              Apr 11 at 9:17










            • $begingroup$
              A transcendental number is a real number that is not algebraic. It is not possible for a real number to be both non algebraic and non transcendental.
              $endgroup$
              – Brady Gilg
              Apr 12 at 17:50













            1












            1








            1





            $begingroup$

            It provides a good random number right until you realize how it was produced, as with many pseudo random number. The irrational (non algebraic and non transcendental) numbers you have chosen are common and so easier guessed then others. I can see no issue with this method provided you choose less commonly seen generators.






            share|cite|improve this answer








            New contributor




            marshal craft is a new contributor to this site. Take care in asking for clarification, commenting, and answering.
            Check out our Code of Conduct.






            $endgroup$



            It provides a good random number right until you realize how it was produced, as with many pseudo random number. The irrational (non algebraic and non transcendental) numbers you have chosen are common and so easier guessed then others. I can see no issue with this method provided you choose less commonly seen generators.







            share|cite|improve this answer








            New contributor




            marshal craft is a new contributor to this site. Take care in asking for clarification, commenting, and answering.
            Check out our Code of Conduct.









            share|cite|improve this answer



            share|cite|improve this answer






            New contributor




            marshal craft is a new contributor to this site. Take care in asking for clarification, commenting, and answering.
            Check out our Code of Conduct.









            answered Apr 11 at 6:20









            marshal craftmarshal craft

            1193




            1193




            New contributor




            marshal craft is a new contributor to this site. Take care in asking for clarification, commenting, and answering.
            Check out our Code of Conduct.





            New contributor





            marshal craft is a new contributor to this site. Take care in asking for clarification, commenting, and answering.
            Check out our Code of Conduct.






            marshal craft is a new contributor to this site. Take care in asking for clarification, commenting, and answering.
            Check out our Code of Conduct.







            • 4




              $begingroup$
              No issue except the gross inefficiency, the fact that you're relying on any adversary not knowing what your algorithm is, the fact that a bad choice of generator could lead to very poor sequence, ...
              $endgroup$
              – David Richerby
              Apr 11 at 9:16






            • 4




              $begingroup$
              By the way, of the numbers suggested in the question, $sqrt2$ is algebraic and $pi$ and $mathrme$ are transcendental.
              $endgroup$
              – David Richerby
              Apr 11 at 9:17










            • $begingroup$
              A transcendental number is a real number that is not algebraic. It is not possible for a real number to be both non algebraic and non transcendental.
              $endgroup$
              – Brady Gilg
              Apr 12 at 17:50












            • 4




              $begingroup$
              No issue except the gross inefficiency, the fact that you're relying on any adversary not knowing what your algorithm is, the fact that a bad choice of generator could lead to very poor sequence, ...
              $endgroup$
              – David Richerby
              Apr 11 at 9:16






            • 4




              $begingroup$
              By the way, of the numbers suggested in the question, $sqrt2$ is algebraic and $pi$ and $mathrme$ are transcendental.
              $endgroup$
              – David Richerby
              Apr 11 at 9:17










            • $begingroup$
              A transcendental number is a real number that is not algebraic. It is not possible for a real number to be both non algebraic and non transcendental.
              $endgroup$
              – Brady Gilg
              Apr 12 at 17:50







            4




            4




            $begingroup$
            No issue except the gross inefficiency, the fact that you're relying on any adversary not knowing what your algorithm is, the fact that a bad choice of generator could lead to very poor sequence, ...
            $endgroup$
            – David Richerby
            Apr 11 at 9:16




            $begingroup$
            No issue except the gross inefficiency, the fact that you're relying on any adversary not knowing what your algorithm is, the fact that a bad choice of generator could lead to very poor sequence, ...
            $endgroup$
            – David Richerby
            Apr 11 at 9:16




            4




            4




            $begingroup$
            By the way, of the numbers suggested in the question, $sqrt2$ is algebraic and $pi$ and $mathrme$ are transcendental.
            $endgroup$
            – David Richerby
            Apr 11 at 9:17




            $begingroup$
            By the way, of the numbers suggested in the question, $sqrt2$ is algebraic and $pi$ and $mathrme$ are transcendental.
            $endgroup$
            – David Richerby
            Apr 11 at 9:17












            $begingroup$
            A transcendental number is a real number that is not algebraic. It is not possible for a real number to be both non algebraic and non transcendental.
            $endgroup$
            – Brady Gilg
            Apr 12 at 17:50




            $begingroup$
            A transcendental number is a real number that is not algebraic. It is not possible for a real number to be both non algebraic and non transcendental.
            $endgroup$
            – Brady Gilg
            Apr 12 at 17:50










            Abhradeep Sarkar is a new contributor. Be nice, and check out our Code of Conduct.









            draft saved

            draft discarded


















            Abhradeep Sarkar is a new contributor. Be nice, and check out our Code of Conduct.












            Abhradeep Sarkar is a new contributor. Be nice, and check out our Code of Conduct.











            Abhradeep Sarkar is a new contributor. Be nice, and check out our Code of Conduct.














            Thanks for contributing an answer to Computer Science Stack Exchange!


            • Please be sure to answer the question. Provide details and share your research!

            But avoid


            • Asking for help, clarification, or responding to other answers.

            • Making statements based on opinion; back them up with references or personal experience.

            Use MathJax to format equations. MathJax reference.


            To learn more, see our tips on writing great answers.




            draft saved


            draft discarded














            StackExchange.ready(
            function ()
            StackExchange.openid.initPostLogin('.new-post-login', 'https%3a%2f%2fcs.stackexchange.com%2fquestions%2f106774%2fcan-we-generate-random-numbers-using-irrational-numbers-like-%25cf%2580-and-e%23new-answer', 'question_page');

            );

            Post as a guest















            Required, but never shown





















































            Required, but never shown














            Required, but never shown












            Required, but never shown







            Required, but never shown

































            Required, but never shown














            Required, but never shown












            Required, but never shown







            Required, but never shown







            Popular posts from this blog

            Wikipedia:Vital articles Мазмуну Biography - Өмүр баян Philosophy and psychology - Философия жана психология Religion - Дин Social sciences - Коомдук илимдер Language and literature - Тил жана адабият Science - Илим Technology - Технология Arts and recreation - Искусство жана эс алуу History and geography - Тарых жана география Навигация менюсу

            Bruxelas-Capital Índice Historia | Composición | Situación lingüística | Clima | Cidades irmandadas | Notas | Véxase tamén | Menú de navegacióneO uso das linguas en Bruxelas e a situación do neerlandés"Rexión de Bruxelas Capital"o orixinalSitio da rexiónPáxina de Bruselas no sitio da Oficina de Promoción Turística de Valonia e BruxelasMapa Interactivo da Rexión de Bruxelas-CapitaleeWorldCat332144929079854441105155190212ID28008674080552-90000 0001 0666 3698n94104302ID540940339365017018237

            What should I write in an apology letter, since I have decided not to join a company after accepting an offer letterShould I keep looking after accepting a job offer?What should I do when I've been verbally told I would get an offer letter, but still haven't gotten one after 4 weeks?Do I accept an offer from a company that I am not likely to join?New job hasn't confirmed starting date and I want to give current employer as much notice as possibleHow should I address my manager in my resignation letter?HR delayed background verification, now jobless as resignedNo email communication after accepting a formal written offer. How should I phrase the call?What should I do if after receiving a verbal offer letter I am informed that my written job offer is put on hold due to some internal issues?Should I inform the current employer that I am about to resign within 1-2 weeks since I have signed the offer letter and waiting for visa?What company will do, if I send their offer letter to another company